kim
Thanks Received: 11
Vinny Gambini
Vinny Gambini
 
Posts: 14
Joined: August 03rd, 2009
 
This post thanked 2 times.
 
 

Q9 - There are far fewer independent

by kim Fri Dec 31, 1999 8:00 pm

9. (A)
Question Type: Assumption


There are fewer independent bookstores now because chain stores have prospered. The lack of independent bookstores has prevented growth in the variety of books available. The author then concludes that the success of the chain stores has been to the detriment of consumers. This conclusion assumes that less book variety = bad for consumers. This logic gap is filled nicely by answer (A).

(B) is not a required assumption. While it may provide support for the premise that states that lack of independent stores has prevented variety in available books, it does not address or resolve the logic gap between a lack of variety and detriment to the book consumer.
(C) is not a required assumption. It may be seeking to equate size of bookstore with variety of books available (larger bookstore = more variety), but this would require another large logical leap, and still wouldn’t address the core issue of whether variety is beneficial to consumers.
(D) is not a required assumption. Again, information about the size of a bookstore gives us no direct information about whether less variety is bad for consumers.
(E) does not validate the conclusion. Whether or not consumers value low prices has nothing to do with whether less variety is bad for consumers.


#officialexplanation
 
PRADEEPCHANDY
Thanks Received: 0
Forum Guests
 
Posts: 13
Joined: June 27th, 2010
 
 
 

Re: Q9 - There are far fewer independent

by PRADEEPCHANDY Wed Jul 07, 2010 1:11 am

why is B not correct?

B removes the logical gap between lack of independent stores and lack of variety in available books

Moreover, if we negate option B - then independent stores = chain stores , then argument breaks down

I think both A and B are correct answers here :)

How should such close answers be handled?
 
giladedelman
Thanks Received: 833
LSAT Geek
 
Posts: 619
Joined: April 04th, 2010
 
This post thanked 2 times.
 
 

Re: PT56, S3, Q9 - there are far fewer independent bookstores

by giladedelman Wed Jul 07, 2010 2:34 pm

Thanks for the question, Pradeep!

Let's think about your claim that if we were to negate answer (B), the argument would fall apart: "Independent bookstores typically do sell the kinds of books that that are available in chain bookstores." Does that mean that independent bookstores = chain stores? No, because they could sell those books and sell a wide variety of other books! (Whole Foods sells the kinds of things that are available at my corner deli -- but they also sell sushi, gourmet coffee, organic produce ...)

But the more important issue is that we're looking for an assumption. The two essential traits of an assumption are that it 1) is unstated and 2) fills in a gap in the argument's logic. Your post refers to "the logical gap between lack of independent stores and lack of variety in available books," but that's not a logical gap: the author doesn't conclude that these are causally connected, he just states it as fact. The only logical gap in this argument is between "lack of variety" and "detriment of book consumers."

I hope that answers your question! Is there anything still tripping you up on this one?
 
shirando21
Thanks Received: 16
Atticus Finch
Atticus Finch
 
Posts: 280
Joined: July 18th, 2012
 
 
 

Re: Q9 - There are far fewer independent

by shirando21 Wed Aug 29, 2012 6:05 pm

but do we put A into conditional logic like this:

a greater variety of readily available--> book consumers better off

Then how do we get

- greater variety-->-book consumers better off (detrimental)

the logic here is:

A-->B, -A
------------
-B
which is invalid

OR gramatically, it makes it sound?
 
shirando21
Thanks Received: 16
Atticus Finch
Atticus Finch
 
Posts: 280
Joined: July 18th, 2012
 
 
 

Re: Q9 - There are far fewer independent

by shirando21 Thu Sep 13, 2012 3:03 pm

Is B reversed?

If B says, chain stores do not sell books that independent books stores sell, would B be correct?

I'd appreciate it very much if any instructor could take a look and discuss with me.
 
patrice.antoine
Thanks Received: 35
Atticus Finch
Atticus Finch
 
Posts: 111
Joined: November 02nd, 2010
 
 
 

Re: Q9 - There are far fewer independent

by patrice.antoine Thu Sep 13, 2012 3:42 pm

shirando21 Wrote:Is B reversed?

If B says, chain stores do not sell books that independent books stores sell, would B be correct?

I'd appreciate it very much if any instructor could take a look and discuss with me.


+1. My thoughts exactly. It is reversed.
 
zainrizvi
Thanks Received: 16
Atticus Finch
Atticus Finch
 
Posts: 171
Joined: July 19th, 2011
 
 
trophy
First Responder
 

Re: PT56, S3, Q9 - there are far fewer independent bookstores

by zainrizvi Thu Apr 25, 2013 1:19 pm

[quote="giladedelman"]

But the more important issue is that we're looking for an assumption. The two essential traits of an assumption are that it 1) is unstated and 2) fills in a gap in the argument's logic. Your post refers to "the logical gap between lack of independent stores and lack of variety in available books," but that's not a logical gap: the author doesn't conclude that these are causally connected, he just states it as fact. The only logical gap in this argument is between "lack of variety" and "detriment of book consumers."/quote]

Really interesting point being made here. I originally saw the shortage of independent bookstores leading to less variety as being an intermediate conclusion, since prevented implies causation. That said I can see it really isn't much of a conclusion because there aren't any supporting premises. Is this a good way to recognize when something is being taken as a premise as opposed to a conclusion? I guess I'm just tripped up on when to things treat things like a premise and when not to.
 
redcobra21
Thanks Received: 4
Elle Woods
Elle Woods
 
Posts: 59
Joined: July 16th, 2013
 
 
 

Re: Q9 - There are far fewer independent

by redcobra21 Fri Sep 13, 2013 5:50 pm

Hey guys

I have a question about how the conditional logic in (A) is valid.

The argument is basically that there has been reduced variety, so this has been to the detriment of book consumers.

If you were to diagram the argument, you'd get something like: ~GV, so ~Better (where GV = greater variety, and Better = being better off).

But if you take a look at (A), it is effectively saying that the argument needs to assume: GV --> Better

How is this an assumption that you need to make? We only know ~GV, but this doesn't tell you anything about GV as is suggested by (A). If (A) is an assumption, it seems like you run into the problem of mistaken negation by saying that because you don't have GV, you will not have Better.

Am I missing something here? I reluctantly picked (B) since that seemed to be the next best choice, but based on the conditional logic, (A) seemed to be absolutely incorrect. I might be wrong so feel free to correct me.

Thanks a lot in advance!!
 
woof90
Thanks Received: 3
Vinny Gambini
Vinny Gambini
 
Posts: 7
Joined: July 07th, 2013
 
This post thanked 3 times.
 
 

Re: Q9 - There are far fewer independent

by woof90 Sun Sep 15, 2013 1:06 pm

redcobra21 Wrote:How is this an assumption that you need to make? We only know ~GV, but this doesn't tell you anything about GV as is suggested by (A). If (A) is an assumption, it seems like you run into the problem of mistaken negation by saying that because you don't have GV, you will not have Better


Hey there! I too struggled with your line of thought even though I guessed (A) to be better than all the other answers. Let me attempt an explanation that might help.

C: The chain bookstores' success has been to the detriment of consumers
P1: The chain bookstores' success reduced the number of independent bookstores
P2: The reduced number of independent bookstores led to less variety than it would've had otherwise.

Before we draw an assumption, we must be careful to note the causal nature of these statements:

C: The chain bookstores' success caused detriment of consumers
P1: The chain bookstores' success caused less # of independent bookstores
P2: The reduced # of independent bookstores caused less variety.

You might say "hold on - are we justified in making these statements causal?" I think so. These causal formulations are well supported by how these premises are phrased in the stimulus ("..success has been to the detriment of ..." / "...has prevented the variety...") THEREFORE, my point is that the assumption also must hinge on a causal, and not conditional, relationship:

Assmp: Less variety caused detriment to consumers

In the LSAT world, a valid cause is the only possible cause without which its effect cannot come about. Since the assumption is that less variety causes badness/detriment, we know that, if we had more variety, it would be relatively better for the consumers! So (A) does not commit the kind of mistaken reversal you see.

Hope this helps!
 
benjaminlpan
Thanks Received: 1
Vinny Gambini
Vinny Gambini
 
Posts: 1
Joined: June 02nd, 2014
 
This post thanked 1 time.
 
 

Re: Q9 - There are far fewer independent

by benjaminlpan Wed Jun 04, 2014 6:27 am

woof90 Wrote:In the LSAT world, a valid cause is the only possible cause without which its effect cannot come about


:shock: On the face of it, that just isn't true.

What tripped me up is while reading (B), my mind's eye inserted a word: "Independent bookstores typically do not sell the [same] kinds of books that are available in chain stores.

As for the debate over whether (A) is problematic with its logic, I think that this may be unfounded.

The argument doesn't require that ~greater variety --> detriment (or ~benefit). It's possible that the use of the word "detriment" is not a logical negation of "better off", but instead a separate claim that the reduced benefit has been to the detriment of consumers. Implicit within all this then, is the assumption that growth of variety --> benefit. Or to put it more plainly, the bit about "as much as it otherwise would have" is rather important: it again re-emphasizes the hidden assumption that growth of variety makes consumers better off. For any lessened benefit to be detrimental at all requires a benefit to be had in the first place, which (A) states precisely.